WBR0371: Difference between revisions

Jump to navigation Jump to search
No edit summary
m (refreshing WBR questions)
 
(4 intermediate revisions by 2 users not shown)
Line 1: Line 1:
{{WBRQuestion
{{WBRQuestion
|QuestionAuthor={{Rim}}
|QuestionAuthor= {{Rim}} (Reviewed by Serge Korjian)
|ExamType=USMLE Step 1
|ExamType=USMLE Step 1
|MainCategory=Pharmacology
|MainCategory=Pharmacology
|SubCategory=Renal
|SubCategory=Gastrointestinal, Renal
|MainCategory=Pharmacology
|MainCategory=Pharmacology
|SubCategory=Renal
|SubCategory=Gastrointestinal, Renal
|MainCategory=Pharmacology
|MainCategory=Pharmacology
|SubCategory=Renal
|SubCategory=Gastrointestinal, Renal
|MainCategory=Pharmacology
|MainCategory=Pharmacology
|MainCategory=Pharmacology
|MainCategory=Pharmacology
|SubCategory=Renal
|MainCategory=Pharmacology
|MainCategory=Pharmacology
|SubCategory=Renal
|SubCategory=Gastrointestinal, Renal
|MainCategory=Pharmacology
|MainCategory=Pharmacology
|SubCategory=Renal
|SubCategory=Gastrointestinal, Renal
|MainCategory=Pharmacology
|MainCategory=Pharmacology
|SubCategory=Renal
|SubCategory=Gastrointestinal, Renal
|MainCategory=Pharmacology
|MainCategory=Pharmacology
|SubCategory=Gastrointestinal, Renal
|MainCategory=Pharmacology
|MainCategory=Pharmacology
|SubCategory=Renal
|MainCategory=Pharmacology
|Prompt=A 67 year old Caucasian male presents to his physician for his annual check-up. The patient has a significant history of hypertension controlled on medications. Appropriate work-up reveals the following parameters and their corresponding activity in the patient’s blood as shown in the figure below.  Which medication is the patient most likely taking to control his blood pressure?
|SubCategory=Gastrointestinal, Renal
 
|Prompt=A 67-year-old man presents to his physician for his annual check-up. The patient has a significant history of hypertension. A hormonal panel reveals the results shown in the table below.  Which medication is the patient most likely taking to control his blood pressure?
[[Image:Nephrology WBR.png]]
|Explanation=The use of either valsartan, an angiotensin II receptor blocker (ARB) or ramipril, an angiotensin II converting enzyme inhibitor (ACEI) will stimulate renin secretion due to the interruption of angiotensin II feedback inhibition.  As such, both medications significantly increase plasma renin concentration.
However, ARB and ACEI act by different mechanisms to inhibit the renin-angiotensin system (RAS).  Because ACEI block the enzyme activity, subsequent products of the enzyme, angiotensin II, are expected to decrease.  In contrary, ARB inhibits the AT1 receptor, thus inhibiting the action of angiotensin II without actually affecting its concentration or production.  As such, concentrations of renin, AT I, and AT II are expected to be increased with the use of ARB.
 
Educational Objective
Although ACEI and ARB inhibit the renin-angiotensin-system (RAS), they work by different mechanisms.  ARB blocks AT I receptor thus causing an elevation of RAS compounds due to inhibition of feedback mechanism.  ACEI inhibits the enzyme activity thus inhibiting conversion from AT I to AT II, leading to an increase in the former and a decrease in the latter.


Reference:
[[Image:WBR0371.png|500px]]
Chen L, Kim SM, Eisner C, et alStimulation of renin secretion by angiotensin II blockade is Gsα-dependentJ Am Soc Nephrol. 2010; 21(8):988-992.
|Explanation=[[File:RAAS_Phys.png|600px]]
The use of either valsartan, an angiotensin II receptor blocker (ARB) or ramipril, an angiotensin II converting enzyme inhibitor (ACEI) will stimulate renin secretion due to the interruption of angiotensin II feedback inhibition.  As such, both medications significantly increase plasma renin concentration. However, ARB and ACEI act by different mechanisms to inhibit the renin-angiotensin system (RAS)Because ACEIs block the the angiotensin converting enzyme, subsequent products of the enzyme such as angiotensin II, are expected to decreaseIn contrast, ARB inhibits the angiotensin II (AT<sub>1</sub> subtype) receptor, thus inhibiting the action of angiotensin II without actually affecting its concentration or production. Given that this patient has elevated concentrations of renin, angiotensin I, and angiotensin II, he is most likely receiving an ARB.
|AnswerA=Valsartan
|AnswerA=Valsartan
|AnswerAExp=Valsartan is an angiotensin II receptor blocker that blocks the AT1 receptor.  With the use of ARB, concentrations of renin, AT I, and AT II are expected to be increased.
|AnswerAExp=Valsartan is an angiotensin II receptor blocker that blocks the AT1 receptor.  With the use of ARB, concentrations of renin, angiotensin I, and angiotensin II are expected to be increased.
|AnswerB=Ramipril
|AnswerB=Ramipril
|AnswerBExp=Ramipril is an ACEI that blocks the conversion of angiotensin I to angiotensin II.  As such, the concentration of angiotensin I is expected to increase, while the concentration of angiotensin II will decrease with ACEI.
|AnswerBExp=Ramipril is an ACEI that blocks the conversion of angiotensin I to angiotensin II.  As such, the concentration of angiotensin I is expected to increase, while the concentration of angiotensin II will decrease with ACEI.
|AnswerC=Acebutolol
|AnswerC=Acebutolol
|AnswerCExp=Acebutolol is a beta-blocker that has a direct effect on sympathetic receptors.  Although RAS is subsequently affected, beta-blockers do not have a direct activity on RAS.
|AnswerCExp=Acebutolol is a beta-blocker that has a direct effect on sympathetic receptors.  Beta-blockers have a small effect on the RAAS by decreasing renin secretion.
|AnswerD=Nifedipine
|AnswerD=Nifedipine
|AnswerDExp=Nifedipine is a dihydropryridine calcium channel blocker that inhibits L-type calcium channel.
|AnswerDExp=Nifedipine is a dihydropryridine calcium channel blocker that inhibits L-type calcium channels in the peripheral blood vessels.
|AnswerE=Diltiazem
|AnswerE=Diltiazem
|AnswerEExp=Diltiazem is a non-dihydropyridine calcium channel blocker that also inhibits L-type calcium channel.
|AnswerEExp=Diltiazem is a non-dihydropyridine calcium channel blocker that also inhibits L-type calcium channels in the heart and in the peripheral blood vessels.
|EducationalObjectives=Although ACEIs and ARBs inhibit the renin-angiotensin-system (RAS), they work by different mechanisms. ACEIs block the the angiotensin converting enzyme, causing angiotensin II concentrations to decrease.  In contrast, ARB inhibits the angiotensin II (AT<sub>1</sub> subtype) receptor, thus inhibiting the action of angiotensin II without actually affecting its concentration or production.
|References=Chen L, Kim SM, Eisner C, et al.  Stimulation of renin secretion by angiotensin II blockade is Gsα-dependent.  J Am Soc Nephrol. 2010; 21(8):988-992.<br>
Image courtesy of the '''Openstax COLLEGE<sup>TM</sup>''' Anatomy & Physiology Textbook, licensed under Creative Commons Attribution 3.0
|RightAnswer=A
|RightAnswer=A
|Approved=No
|WBRKeyword=RAAS, Renin Angiotensin aldosterone system, Renin, ACE inhibitor, ARB, Angiotensin-converting enzyme,  Angiotensin receptor blocker
|Approved=Yes
}}
}}

Latest revision as of 00:19, 28 October 2020

 
Author [[PageAuthor::Rim Halaby, M.D. [1] (Reviewed by Serge Korjian)]]
Exam Type ExamType::USMLE Step 1
Main Category MainCategory::Pharmacology
Sub Category SubCategory::Gastrointestinal, SubCategory::Renal
Prompt [[Prompt::A 67-year-old man presents to his physician for his annual check-up. The patient has a significant history of hypertension. A hormonal panel reveals the results shown in the table below. Which medication is the patient most likely taking to control his blood pressure?

]]

Answer A AnswerA::Valsartan
Answer A Explanation AnswerAExp::Valsartan is an angiotensin II receptor blocker that blocks the AT1 receptor. With the use of ARB, concentrations of renin, angiotensin I, and angiotensin II are expected to be increased.
Answer B AnswerB::Ramipril
Answer B Explanation AnswerBExp::Ramipril is an ACEI that blocks the conversion of angiotensin I to angiotensin II. As such, the concentration of angiotensin I is expected to increase, while the concentration of angiotensin II will decrease with ACEI.
Answer C AnswerC::Acebutolol
Answer C Explanation AnswerCExp::Acebutolol is a beta-blocker that has a direct effect on sympathetic receptors. Beta-blockers have a small effect on the RAAS by decreasing renin secretion.
Answer D AnswerD::Nifedipine
Answer D Explanation AnswerDExp::Nifedipine is a dihydropryridine calcium channel blocker that inhibits L-type calcium channels in the peripheral blood vessels.
Answer E AnswerE::Diltiazem
Answer E Explanation AnswerEExp::Diltiazem is a non-dihydropyridine calcium channel blocker that also inhibits L-type calcium channels in the heart and in the peripheral blood vessels.
Right Answer RightAnswer::A
Explanation [[Explanation::

The use of either valsartan, an angiotensin II receptor blocker (ARB) or ramipril, an angiotensin II converting enzyme inhibitor (ACEI) will stimulate renin secretion due to the interruption of angiotensin II feedback inhibition. As such, both medications significantly increase plasma renin concentration. However, ARB and ACEI act by different mechanisms to inhibit the renin-angiotensin system (RAS). Because ACEIs block the the angiotensin converting enzyme, subsequent products of the enzyme such as angiotensin II, are expected to decrease. In contrast, ARB inhibits the angiotensin II (AT1 subtype) receptor, thus inhibiting the action of angiotensin II without actually affecting its concentration or production. Given that this patient has elevated concentrations of renin, angiotensin I, and angiotensin II, he is most likely receiving an ARB.
Educational Objective: Although ACEIs and ARBs inhibit the renin-angiotensin-system (RAS), they work by different mechanisms. ACEIs block the the angiotensin converting enzyme, causing angiotensin II concentrations to decrease. In contrast, ARB inhibits the angiotensin II (AT1 subtype) receptor, thus inhibiting the action of angiotensin II without actually affecting its concentration or production.
References: Chen L, Kim SM, Eisner C, et al. Stimulation of renin secretion by angiotensin II blockade is Gsα-dependent. J Am Soc Nephrol. 2010; 21(8):988-992.
Image courtesy of the Openstax COLLEGETM Anatomy & Physiology Textbook, licensed under Creative Commons Attribution 3.0]]

Approved Approved::Yes
Keyword WBRKeyword::RAAS, WBRKeyword::Renin Angiotensin aldosterone system, WBRKeyword::Renin, WBRKeyword::ACE inhibitor, WBRKeyword::ARB, WBRKeyword::Angiotensin-converting enzyme, WBRKeyword::Angiotensin receptor blocker
Linked Question Linked::
Order in Linked Questions LinkedOrder::